Une somme :)

Bonjour! j'ai créé un petit exercice.

Calculer $\quad\displaystyle \sum_{n=0}^{\infty} (-1)^n \frac{1}{(2n+1)^3}$.
Je suis donc je pense 

Réponses

  • Bonjour.

    Le résultat est $\frac{\pi^3}{32}$.

    À bientôt.

    Cherche livres et objets du domaine mathématique :

    Intégraphes, règles log et calculateurs électromécaniques.

  • Cette somme est non triviale. Voir fonction bêta de Dirichlet.

    PS:
    Si je me souviens bien on peut faire un lien entre cette somme et $\zeta(4)$. Ce qui fait que si son connait une expression de $\zeta(4)$ on connait une expression de cette somme.
  • Dreamer c'est exact :-) mais... peux-tu le démontrer ? (moi je l'ai fait avec le théorème des résidus pour les sommes).
    Je suis donc je pense 
  • Ça se fait avec les séries de Fourier, développement de $x^2$ sur $[-\pi,\pi]$, et de sa primitive.
    Il me semble que Jandri avait donné une solution « dé-fouriérisée », mais impossible de la retrouver.
    Bonne après-midi.
    Fr. Ch.
  • Quentino37, tu as demandé de calculer.

    De plus, Fin de partie à donné le lien avec la fonction Bêta de Dirichlet qui est selon moi le chemin le plus direct.

    Et pour finir, c'est en partant de sommations "à la Euler" que je suis arrivé au résultat et ce n'est pas une bonne façon de faire, je préfère donc ne pas la poster.

    À bientôt.

    Cherche livres et objets du domaine mathématique :

    Intégraphes, règles log et calculateurs électromécaniques.

  • Pour moi c'est plus facile à retrouver (j'ai pris des notes !)

    C'était il y a presque 5 ans, c'est bien caché dans le fil : Sudoku des intégrales convergentes
  • \begin{align}\beta(3)&=\sum_{n=0}^\infty \frac{(-1)^n}{(2n+1)^3}\\
    &=\frac{1}{2}\int_0^1 \frac{\ln^2 x}{1+x^2}dx\\
    &\overset{y=\frac{1}{x}}=\frac{1}{2}\int_1^\infty \frac{\ln^2 y}{1+y^2}dy\\
    &=\frac{1}{4}\int_0^\infty \frac{\ln^2 x}{1+x^2}dx\\
    K&=\int_0^\infty\int_0^\infty \frac{\ln^2 (xy)}{(1+x^2)(1+y^2)}dxdy\\
    &\overset{u(x)=xy}=\int_0^\infty\int_0^\infty \frac{y\ln^2 u}{(1+y^2)(u^2+y^2)}dudy\\
    &=\frac{1}{2}\int_0^\infty \left[\frac{\ln\left(\frac{1+y^2}{u^2+y^2}\right)}{u^2-1}\right]_{y=0}^{y=\infty} \ln^2 udu\\
    &=\int_0^\infty \frac{\ln^3 u}{u^2-1}du\\
    &=\int_0^1 \frac{\ln^3 u}{u^2-1}du+\underbrace{\int_1^\infty \frac{\ln^3 u}{u^2-1}du}_{z=\frac{1}{u}}\\
    &=2\int_0^1 \frac{\ln^3 u}{u^2-1}du\\
    &=-\int_0^1 \frac{\ln^3 u}{1+u}du-\int_0^1 \frac{\ln^3 u}{1-u}du\\
    &=-\int_0^1 \frac{\ln^3 u}{1-u}du+\underbrace{\int_0^1 \frac{2u\ln^3 u}{1-u^2}du}_{z=u^2}-\int_0^1 \frac{\ln^3 u}{1-u}du\\
    &=-\frac{15}{8}\int_0^1 \frac{\ln^3 u}{1-u}du\\
    &=-\frac{15}{8}\times -6\zeta(4)\\
    &=\frac{45}{4}\zeta(4)\\
    K&=2\int_0^\infty\int_0^\infty \frac{\ln^2 x}{(1+x^2)(1+y^2)}dxdy+2\bigg(\underbrace{\int_0^\infty \frac{\ln x}{1+x^2}dx}_{=0}\bigg)^2\\
    &=4\pi\beta(3)\\
    \end{align}

    Donc, $\displaystyle\boxed{\beta(3)=\dfrac{45\zeta(4)}{16\pi}}$

    Si on sait que $\zeta(4)=\dfrac{\pi^4}{90}$ alors on a $\boxed{\beta(3)=\dfrac{\pi^3}{32}}$
    NB: On a $\displaystyle \int_0^1 \dfrac{\ln^3 x}{1-x}dx=-6\zeta(4),\zeta(4)=\sum_{n=1}^\infty \dfrac{1}{n^4}$.
  • A partir de $\frac{1}{4}\int_0^{\infty}\frac{(\log x)^2}{1+x^2}dx=\frac{1}{32}\int_0^{\infty}\frac{(\log v)^2}{\sqrt{v}(1+v)}dv$ et de $\int_0^{\infty}\frac{v^{s-1}}{1+v}dv=\frac{\pi}{\sin{\pi s}},$ c'est de la routine car il suffit de faire $s=1/2$ dans
    $$\frac{d^2}{ds^2}\int_0^{\infty}\frac{v^{s-1}}{1+v}dv=\int_0^{\infty}\frac{(\log v)^2v^{s-1}}{1+v}dv$$
  • Je rappelle l'utilisation des séries de Fourier.

    $\bullet $ Soit la fonction $2\pi $- périodique $f$ telle que : $f(x)=x^{2}$ pour $x\in \lbrack -\pi ,\pi \lbrack $. C'est une fonction continue et $\mathcal{C}^{1}$ par morceaux, d'où le développement en série de Fourier, pour tout $x\in \mathbb{R}$ : $\displaystyle f(x)=\frac{\pi ^{2}}{3}+4\overset{+\infty }{\underset{n=1}{\sum }}(-1)^{n}\frac{\cos nx}{n^{2}}$, et en particulier pour $x\in \lbrack -\pi ,\pi ]$ : $\displaystyle x^{2}=\frac{\pi ^{2}}{3}+4\overset{+\infty }{%
    \underset{n=1}{\sum }}(-1)^{n}\frac{\cos nx}{n^{2}}$, soit : $ \displaystyle \frac{1}{4}%
    (x^{2}-\frac{\pi ^{2}}{3})=\overset{+\infty }{\underset{n=1}{\sum }}(-1)^{n}%
    \frac{\cos nx}{n^{2}}$.

    $\bullet $ Déjà en faisant $x:=\pi $, il vient : $\displaystyle \frac{\pi ^{2}}{6}=\overset{+\infty }{\underset{n=1}{\sum }}\frac{1}{n^{2}} =\zeta(2)$. C'est une des multiples démonstrations de cette formule, que nous avons souvent évoquées sur ce forum.

    $\bullet $ La convergence de cette série de fonctions étant normale sur $%
    \mathbb{R}$ tout entier, on a, pour $x\in \lbrack -\pi ,\pi ]$ :
    $\displaystyle \frac{1}{4}\int_{0}^{x}(t^{2}-\frac{\pi ^{2}}{3})dt=\overset{+\infty }{%
    \underset{n=1}{\sum }}\int_{0}^{x}(-1)^{n}\frac{\cos nt}{n^{2}}dt$, soit :
    $\displaystyle \frac{1}{4}(-\frac{x^{3}}{3}+\frac{\pi ^{2}}{3}x)=\overset{+\infty }{%
    \underset{n=1}{\sum }}(-1)^{n-1}\frac{\sin nx}{n^{3}}$.
    En faisant $x:=\frac{%
    \pi }{2}$, il vient : $\displaystyle \frac{\pi ^{3}}{32}=\overset{+\infty }{\underset{q=1}{%
    \sum }}\frac{\sin ((2q-1)\frac{\pi }{2})}{(2q-1)^{3}}=\overset{+\infty }{%
    \underset{q=1}{\sum }}\frac{(-1)^{q-1}}{(2q-1)^{3}}=\beta (3)$.

    $\bullet $ On peut continuer les intégrations, pour $x\in \lbrack -\pi ,\pi ]$ : $ \displaystyle \frac{1}{12}\int_{0}^{x}(-t^{3}+\pi ^{2}t)dt=\overset{+\infty }{%
    \underset{n=1}{\sum }}\int_{0}^{x}(-1)^{n-1}\frac{\sin nt}{n^{3}}dt$, soit $\displaystyle
    \frac{1}{12}(-\frac{x^{4}}{4}+\pi ^{2}\frac{x^{2}}{2})=\overset{+\infty }{%
    \underset{n=1}{\sum }}(-1)^{n-1}\frac{-\cos nx+1}{n^{4}}$.
    En faisant $x:=\pi $, il vient : $\displaystyle \frac{\pi ^{4}}{48}=\overset{+\infty }{\underset{n=1}{%
    \sum }}\frac{1+(-1)^{n-1}}{n^{4}}=\overset{+\infty }{\underset{q=1}{\sum }}%
    \frac{2}{(2q-1)^{4}}$,
    et l'on en déduit facilement $\displaystyle \zeta (4)=\overset{%
    +\infty }{\underset{n=1}{\sum }}\frac{1}{n^{4}}=\frac{\pi ^{4}}{90}$, et $\displaystyle
    \eta (4)=\overset{+\infty }{\underset{n=1}{\sum }}\frac{(-1)^{n-1}}{n^{4}}=%
    \frac{7\pi ^{4}}{720}$.

    $\bullet $ On a donc pour $x\in \lbrack -\pi ,\pi ]$ : $\displaystyle \frac{x^{4}}{%
    48}-\pi ^{2}\frac{x^{2}}{24}+\frac{7\pi ^{4}}{720}=\overset{+\infty }{%
    \underset{n=1}{\sum }}(-1)^{n-1}\frac{\cos nx}{n^{4}}$.
    Une nouvelle intégration donne, pour $x\in \lbrack -\pi ,\pi ]$ : $\displaystyle \frac{x^{5}}{240}-\pi ^{2}%
    \frac{x^{3}}{72}+\frac{7\pi ^{4}}{720}x=\overset{+\infty }{\underset{n=1}{%
    \sum }}(-1)^{n-1}\frac{\sin nx}{n^{5}}$.
    En faisant encore $x:=\frac{\pi }{2%
    }$, on obtient la valeur de $\displaystyle \beta (5)=\overset{+\infty }{\underset{q=1}{%
    \sum }}\frac{(-1)^{q-1}}{(2q-1)^{5}}=\frac{5\pi ^{5}}{1536}$ donnée par Jandri.

    $\bullet $ Les séries de Fourier sont vraiment une usine à formules ! Dommage qu'on en ait privé les taupins...

    RÉFÉRENCES
    Tolstov, Georgi P., Fourier Series, Dover, 1976, p. 25.
    Ramis, Deschamps, Odoux, Exercices d'Analyse 2 (bleu), Masson 1985, p. 104.
    Aubonnet, Guinin, Oral, exercices résolus, « Crus » 1988-89, Bréal 1989, p. 120.
    http://www.les-mathematiques.net/phorum/read.php?4,1236063,1313706#msg-1313706
  • Bonjour Chaurien, tu écris avec quel logiciel ?
  • Chaurien cite Tolstoï.
    A demon  wind propelled me east of the sun
  • Hunter, qu'est-ce qui te fait croire que j'écris avec un logiciel ?
  • Je rappelle aussi le calcul de cette somme de série au moyen des résidus.

    $\bullet$ La fonction de variable complexe $f(z)=\frac 1{z^3 \cos z}$ a pour pôles $0$ et $\frac {\pi}2 +n \pi$, $n \in \mathbb Z$.

    $\bullet$ Chaque pôle $\frac {\pi}2 +n \pi$, $n \in \mathbb Z$, est simple, d'où le résidu : $\textrm{Res}(f, \frac {\pi}2 +n \pi)= \lim_{h \rightarrow 0} hf(\frac {\pi}2 +n \pi+h)=\frac{8(-1)^{n+1}}{(2n+1)^{3}\pi ^{3}}$.
    De plus, $f(z)=\frac{1}{z^{3}\cos z}=\frac{1}{z^{3}(1-\frac{z^{2}}{2}+\frac{z^{4}}{24}%
    +...)}=\frac{1}{z^{3}}(1+\frac{z^{2}}{2}+\frac{5z^{4}}{24}+...)=\frac{1}{%
    z^{3}}+\frac{1}{2z}+\frac{5z}{24}+...$, d'où : $\textrm{Res}(f,0)=\frac{1}{2}$.

    $\bullet$ Pour $n\in \mathbb{N}^{\ast }$, soit le cercle $\Gamma _{n}=\{z~|~\left\vert z\right\vert =n\pi \}$. Le théorème des résidus dit que :
    $~~~~~~~~~~~~~~~~~~~~~\displaystyle \frac{1}{2\pi i}\int_{\Gamma _{n}}f(z)dz
    =\textrm{Res}(f,0)+\underset{k=-n}{\overset{k=n-1}{\sum }} \textrm{Res}(f,\frac{\pi }{2}+k\pi )$
    $~~~~~~~~~~~~~~~~~~~~~~~~~~~~~~~~~~\displaystyle =\frac{1}{2}+\underset{k=-n}{\overset{k=-1}{\sum }}\frac{8(-1)^{k+1}}{(2k+1)^{3}\pi ^{3}}+\underset{k=0}{\overset{k=n-1}{\sum }}\frac{8(-1)^{k+1}}{%
    (2k+1)^{3}\pi ^{3}}=\frac{1}{2}-\frac{2}{\pi ^{3}}\underset{h=1}{\overset{n}{%
    \sum }}\frac{8(-1)^{h-1}}{(2h-1)^{3}}$.

    $\bullet$ Pour $n \in \mathbb N^*$ soit $R_n=n\pi$, alors : $\displaystyle \int_{\Gamma _{n}}f(z)dz=\int_{\Gamma _{n}}\frac{dz}{z^{3}\cos z}%
    =\int_{0}^{2\pi }\frac{R_{n}ie^{i\theta }d\theta }{R_{n}^{3}e^{3i\theta
    }\cos (R_{n}e^{i\theta })}=\int_{0}^{2\pi }\frac{ie^{-2i\theta }d\theta }{%
    R_{n}^{2}\cos (R_{n}e^{i\theta })}$.

    Or, on peut prouver que si $|z|=R_n=n\pi$, alors : $\left\vert \cos z\right\vert >\frac{1}{2}$ (en fait $\left\vert \cos z\right\vert >\frac{99}{100}$, mais n'importe quel minorant constant strictement positif fait l'affaire).

    D'où : $\displaystyle \left\vert \int_{\Gamma _{n}}f(z)dz\right\vert \leq \int_{0}^{2\pi
    }\left\vert \frac{ie^{-2i\theta }d\theta }{R_{n}^{2}\cos (R_{n}e^{i\theta })}\right\vert =\int_{0}^{2\pi }\frac{d\theta }{%
    R_{n}^{2}\left\vert \cos (R_{n}e^{i\theta })\right\vert }\leq \frac{4\pi }{%
    R_{n}^{2}}$.
    Ce qui implique : $\displaystyle \lim_{n\rightarrow +\infty }\frac{1}{2\pi i}\int_{\Gamma _{n}}f(z)dz=0$, soit : $\displaystyle \frac{1}{2}-\frac{2}{\pi ^{3}}\underset{h=1}{\overset{+\infty }{\sum }}%
    \frac{8(-1)^{h-1}}{(2h-1)^{3}}=0$, CQFD.

    Bonne journée de ce premier juillet 2021, et bonnes vacances, notamment à mes collègues professeurs, qui les ont bien méritées.
    Fr. Ch.
  • j'ai remarqué une façon un peu spéciale d'écrire..
  • Complément. Si $n \in \mathbb N^*$, si $z \in \mathbb C$, si $|z|=n \pi$, alors $|\cos z| >\frac 12$ (ou $|\cos z| >\frac {99}{100}$).
    Démonstration. Soit $z=x+iy$, $x \in \mathbb R$, $y \in \mathbb R$. Alors $|\cos z|^2=\cos^2 x+ \sinh^2 y$.
    Comme $|\cos z|= |\cos(-z)|= |\cos \overline{z}|$, il suffit de prouver l'inégalité pour le premier quadrant : $ x\ge 0$ et $y \ge 0$.
    Prenons $\alpha \in ]0,\frac {\pi}2[$, et distinguons deux cas.
    - Si $n \pi - \alpha \le x \le n \pi$, alors $|\cos z| \ge |\cos x| \ge \cos \alpha$.
    - Si $0 \le x \le n \pi - \alpha $, alors $y^2=(n \pi)^2-x^2 \ge (n \pi)^2-(n \pi - \alpha)^2 \ge \alpha (2 \pi - \alpha) $, d'où : $|\cos z| \ge \sinh y \ge \sinh \sqrt {\alpha (2 \pi - \alpha)}$.
    On peut alors donner à $\alpha$ telle ou telle valeur dans $]0,\frac {\pi}2[$, par exemple $\frac {\pi}4$, ou autre, on aura un minorant de $|\cos z| $ strictement positif indépendant de $n$, et c'est ce qui compte.
    Le camarade WolframAlpha me dit que les deux courbes : $\alpha \mapsto \cos \alpha$ et $\alpha \mapsto \sinh \sqrt {\alpha (2 \pi - \alpha)}$, pour $\alpha \in [0, \frac {\pi}2]$, se croisent en $\alpha \simeq 0,124569$ et donnent la valeur $0,99225$, ce qui démontre l'inégalité annoncée.
  • Hunter, veux-tu expliquer en quoi ma façon d'écrire serait « spéciale » ?
  • Pour les résidus, inutile de refaire à chaque fois la démonstration. Les corollaires existent et peuvent être utilisés directement.

    Par exemple, dans le cas qui nous occupe, il y a celui-ci, entre autres :

    Corollaire. Si $P$ et $Q$ sont deux polynômes tels que $\deg Q \geqslant \deg P + 2$ et si $a_1,\dotsc,a_r$ sont les zéros distincts de $Q$, alors
    $$\sum_{n \in \mathbb{Z}} (-1)^n \frac{P(n)}{Q(n)} = - \sum_{k=1}^r \ \underset{s=a_k}{\textrm{Res}} \left( \frac{\pi f(s)}{\sin (\pi s)} \right).$$
    Pour une série non alternée, remplacer le sinus du dénominateur par une tangente.
  • Noix de Totos, je ne fais pas souvent de calcul de résidus, et je n'en ai pas fait professionnellement, sauf une fois lors d'un cours d'été dans une boîte privée. Alors je préfère détailler, ce qui peut être utile à des lecteurs aussi peu familiers que moi avec cette question.
    On peut prendre un autre contour d'intégration, par exemple un carré, mais j'aime bien mon cercle, avec sa minoration de $|\sin z|$.
    Et merci pour le joli $\textrm{Res}$ que je ne connaissais pas et que j'ai piqué.
    Bonne soirée.
    Fr. Ch.
  • Le calcul que j'ai présenté s'applique sans investissement supplémentaire pour obtenir $\displaystyle \beta (5)=\overset{+\infty }{\underset{n=1}{\sum }}\frac{(-1)^{n-1}}{(2n-1)^{5}}$.
    On prend $f(z)=\frac 1{z^5 \cos z}=\frac{1}{z^{5}}+\frac{1}{2z^3}+\frac{5}{24z}+...$, d'où $\textrm{Res}(f,0)=\frac{5}{24}$.
    De plus $\textrm{Res}(f, \frac {\pi}2 +n \pi)= \lim_{h \rightarrow 0} hf(\frac {\pi}2 +n \pi+h)=\frac{32(-1)^{n+1}}{(2n+1)^{5}\pi ^{5}}$.
    On a comme précédemment : $\displaystyle \frac{1}{2\pi i}\int_{\Gamma _{n}}f(z)dz=\textrm{Res}(f,0)+\underset{k=-n}{\overset{k=n-1}{\sum }} \textrm{Res}(f,\frac{\pi }{2}+k\pi )=\frac{5}{24}-\frac{2}{\pi ^{5}}\underset{h=1}{\overset{n}{\sum }}\frac{32(-1)^{h-1}}{(2h-1)^5}$.
    Cette intégrale tend vers $0$ en $+ \infty$, pour la même raison que la précédente, et l'on conclut immédiatement : $\displaystyle \overset{+\infty }{\underset{h=1}{\sum }}\frac{(-1)^{h-1}}{(2h-1)^{5}}=\frac {5 \pi^5}{1536}$.
    C'est vraiment merveilleux, la théorie de la variable complexe : on devrait en mettre les rudiments au programme de Math Spé. Bon, je sais, ce n'est pas demain la veille...
    Bonne soirée.
    Fr. Ch.
  • Chaurien a écrit:
    C'est vraiment merveilleux, la théorie de la variable complexe : on devrait en mettre les rudiments au programme de Math Spé. Bon, je sais, ce n'est pas demain la veille...
    Étant donné qu'à compter de la rentrée prochaine, une partie non négligeable des élèves rentrant en classe prépa n'aura jamais connu les complexes avant d'y entrer, il me parait difficile de faire suffisamment de choses pour atteindre ces jolis résultats, pourtant pas si loin des séries entières...
  • Je comprends ta démarche, mais la mienne n'en est pas moins utile pour le lecteur débutant : on a aujourd'hui, et ce depuis Lindelöf, une foultitudes de résultats établis à partir du calcul des résidus.

    Voici donc mon conseil :

    (i) Effectuer une preuve de ce type une fois ou deux pour voir comment ça marche ;

    (ii) Accroître ses connaissances et élargir les corollaires du théorème des résidus à un grand nombre de situations. Cette étape, pas toujours évidente à faire, nécessite une recherche accrue parmi les bons livres et les articles de recherche adéquats.

    Edit. Je répondais au message de Chaurien ci-dessus.
  • En effet, bisam, je plaisantais. J'avais fait un article d'initiation pour les prépas : Calcul de sommes de séries numériques au moyen du théorème des résidus, Sigma, Revue de mathématiques et de sciences physiques, n° 2, juin 1996, CPR Tanger, pp. 107-114. Je traitais notamment la somme des $\frac 1{x_n^2}$, où $x_n$ est la $n$-ème solution positive de l’équation $\tan x=x$, question dont nous parlons de temps en temps sur ce forum. Cet article pourrait être utile aujourd'hui encore, mais il doit être difficile à trouver.
  • L'appétit vient en mangeant. La ressemblance entre le calcul de $\displaystyle \beta (3)=\overset{+\infty }{\underset{n=1}{\sum }}\frac{(-1)^{n-1}}{(2n-1)^{3}}$ et de $\displaystyle \beta (5)=\overset{+\infty }{\underset{n=1}{\sum }}\frac{(-1)^{n-1}}{(2n-1)^{5}}$ me fait penser à la possibilité de calculer en général $ \displaystyle \beta (2p+1)=\overset{+\infty }{\underset{n=1}{\sum }}\frac{(-1)^{n-1}}{(2n-1)^{2p+1}} $, $p \in \mathbb N^*$.
    La fonction $z \mapsto \frac1{\cos z}$ est connue comme fonction sécante, elle existait en France aussi avant qu'une décision stupide du début du XXème siècle ne la supprime de l'enseignement, bêtise que n'ont pas faite les Anglo-Saxons. Cette fonction a un développement en série entière qui s'exprime au moyen des nombres d'Euler : $\displaystyle \frac 1{\cos z}=\overset{+\infty }{\underset{n=0}{\sum }}E_{2n} \frac{z^{2n}}{(2n)!}$. On considère donc la fonction $f(z)=\frac 1{z^{2p+1} \cos z}$, et l'on a son résidu au pôle $0$ qui est $\frac {E_{2p}}{(2p)!}$. Le reste se fait comme précédemment.
    Bonne nuit.
    Fr. Ch.
  • On peut utiliser la même méthode pour calculer $\beta(2n+1)$ que celle utilisée dans mon message plus haut.

    Mais il faut considérer l'intégrale $\displaystyle K_n=\int_0^\infty \int_0^\infty \frac{\ln(xy)^{2n}}{(1+x^2)(1+y^2)}dxdy$
    et si je me souviens bien on obtiendra une relation de récurrence. Comme dans le calcul mentionné on aura besoin de connaître la valeur de $\zeta(2n+2)$ (et probablement de $\zeta(2k+2)$ pour $0\leq k\leq n$ )
  • Je reviens sur le minorant de $|\cos z|$ de mon précédent message :
    http://www.les-mathematiques.net/phorum/read.php?4,2266222,2269828#msg-2269828
    Ce $0,99$ m'a turlupiné, et il me semblait qu'on devrait plutôt trouver $1$. Prenant les choses différemment, je crois avoir démontré que $|\cos z| \ge |\cos |z| |$. C'est sans doute très connu, mais je ne m'en étais jamais aperçu.
  • Pour répondre sur ce point précis de la minoration de $\left| \cos z \right|$, on peut faire mieux : par exemple
    $$\forall z \in \mathbb{C}, \quad \left| \cos z \right| \geqslant \left| \textrm{sh} \left( \textrm{Im} \, z \right) \right|.$$

    M. Abramovitz & I. A. Stegun, Handbook of Mathematical Functions, New-York, 1965, inégalité 4.3.84 page 75.
  • Oui, Noix de Totos, je connais bien ce vénérable monstre Abramovitz & Stegun depuis longtemps, et au fait je ne le consulte pas assez.
    L'inégalité que tu donnes est présente dans mon message ci-dessus puisqu'il y est dit que si $z=x+iy$, $x$ et $y$ réels, alors $|\cos z|^2=\cos^2 x+ \sinh^2 y$. Mais à elle seule elle ne peut prouver que si $|z|=n \pi$, alors $|\cos z | \ge 1$, ce que donne la minoration de mon dernier message.
    Bonne soirée.
    Fr. Ch.
  • Dans ce cas, une démonstration de cette inégalité pourrait être intéressante.
  • Je ne connaissais pas l'inégalité donnée par Chaurien : $|\cos z| \ge |\cos |z| |$

    Elle se démontre très simplement en étudiant $f(y)=\cos^2 x+ \sinh^2 y - \cos^2 \sqrt{x^2+y^2}$.

    On a $f(0)=0$ et par parité on peut se limiter à $y\geq0$.

    $f'(y)=\sinh (2y) + \dfrac y{\sqrt{x^2+y^2}}\sin( 2\sqrt{x^2+y^2})\geq0$ puisque $|\sin t|\leq t$ et $\sinh t\geq t$ pour $t\geq0$.
  • Voici ma démonstration de l’inégalité : $|\cos z| \ge |\cos |z| |$, pour $z\in \mathbb{C}$.
    Comme $\left\vert \cos z\right\vert =\left\vert \cos (-z)\right\vert =\left\vert\cos \overline{z}\right\vert $, il suffit de la démontrer pour $ \textrm{Re}~ z \ge 0$ et $ \textrm{Im} ~z \ge 0$.

    $\bullet $ Lemme. Si $x\in \mathbb{R}$, $y\in \mathbb{R}$, $x>0$, $y>0$, alors : $\frac{\left\vert \sin x\right\vert }{x}<1<\frac{\sinh y}{y}$.

    $\bullet $ Soit $a\in \mathbb{R}$, $a>0$. Pour $x\in \lbrack 0,a]$, soit $f_{a}(x)=\cos x+\cosh \sqrt{a^{2}-x^{2}}$.
    D'après le lemme, on a, pour $x\in ]0,a[$ : $f_{a}^{~\prime }(x)=-x(\frac{\sin x}{x}+\frac{\sinh \sqrt{a^{2}-x^{2}}}{\sqrt{a^{2}-x^{2}}})<0$.
    La fonction $f_a$ est donc strictement décroissante sur $[0,a]$, et l'on en déduit : $f_{a}(x)\geq f_{a}(a)=1+\cos a$.

    $\bullet $ Soit $R\in \mathbb{R}$, $R>0$, et soit $z\in \mathbb{C}$, $\left\vert z\right\vert =R$. Soit $z=x+iy$, $x\in \mathbb{R}$, $y\in \mathbb{R}$, $x\geq 0$, $y\geq 0$.
    $~~~~~~~~$Alors : $|\cos z|^{2}=\cos^2x+\sinh^2 y=\cos^2x+\sinh^2 \sqrt{R^{2}-x^{2}}=\frac{1}{2}f_{2R}(2x)$.
    D'où : $|\cos z|^{2}\geq \frac{1}{2}f_{2R}(2R)=\frac{1}{2}(1+\cos 2R)=\cos ^{2}R=(\cos |z|)^{2}$.

    $\bullet $ On en déduit que si $n \in \mathbb N^*$, si $z \in \mathbb C$, si $|z|=n \pi$, alors$|\cos z| \ge 1$, et là c'est le meilleur minorant possible. Au départ, c’était pour un calcul de limite d'intégrale sur un contour, et on a vu que tout minorant constant strictement positif faisait l'affaire, mais c'était juste par curiosité.

    $\bullet $ On démontre de même que $|\sin z| \ge |\sin |z| |$.
    Corollaire : si $n \in \mathbb N$, si $z \in \mathbb C$, si $|z|=\frac {\pi}2+n \pi$, alors$|\sin z| \ge 1$.

    Bonne journée quoique brumeuse.
    Fr. Ch.
    05/07/2021
  • Et tiens, pendant qu'on y est, en vue d'autres calculs de résidus, je crois avoir démontré que si $n \in \mathbb N^*$, si $z \in \mathbb C$, si $|z|= n \pi$, alors $|\tan z| < 1,032$. Cette constante n'est pas la meilleure possible (déjà ce n'est qu'une valeur approchée), mais je suis certain qu'on ne peut la remplacer par $1$, contrairement au $0,99$ évoqué plus haut pour le cosinus. Les maths, c'est varié.
Connectez-vous ou Inscrivez-vous pour répondre.